Difference between revisions of "1982 AHSME Problems/Problem 29"

(Created page with "Answer is 69")
 
(One intermediate revision by one other user not shown)
Line 1: Line 1:
Answer is 69
+
==Problem==
 +
Let <math>x,y</math>, and <math>z</math> be three positive real numbers whose sum is <math>1</math>. If no one of these numbers is more than twice any other, then the minimum possible value of the product <math>xyz</math> is
 +
 
 +
<math>\textbf{(A)}\ \frac{1}{32}\qquad \textbf{(B)}\ \frac{1}{36}\qquad \textbf{(C)}\ \frac{4}{125}\qquad \textbf{(D)}\ \frac{1}{127}\qquad \textbf{(E)}\ \text{none of these}</math>
 +
==Solution==
 +
The answer is A, 1/32, as obtained by (1/4) * (1/4) * (1/2).
 +
==See Also==
 +
{{AHSME box|year=1982|num-b=28|num-a=30}}
 +
 
 +
{{MAA Notice}}

Revision as of 16:41, 17 June 2021

Problem

Let $x,y$, and $z$ be three positive real numbers whose sum is $1$. If no one of these numbers is more than twice any other, then the minimum possible value of the product $xyz$ is

$\textbf{(A)}\ \frac{1}{32}\qquad \textbf{(B)}\ \frac{1}{36}\qquad \textbf{(C)}\ \frac{4}{125}\qquad \textbf{(D)}\ \frac{1}{127}\qquad \textbf{(E)}\ \text{none of these}$

Solution

The answer is A, 1/32, as obtained by (1/4) * (1/4) * (1/2).

See Also

1982 AHSME (ProblemsAnswer KeyResources)
Preceded by
Problem 28
Followed by
Problem 30
1 2 3 4 5 6 7 8 9 10 11 12 13 14 15 16 17 18 19 20 21 22 23 24 25 26 27 28 29 30
All AHSME Problems and Solutions


The problems on this page are copyrighted by the Mathematical Association of America's American Mathematics Competitions. AMC logo.png